Significance of assumption in competition inequality questions











up vote
1
down vote

favorite
1












Refer to the problem below (IMO 2009 Shortlist)




Let $a, b, c$ be positive real numbers such that $$frac{1}{a} + frac{1}{b} + frac{1}{c} = a + b+ c$$
Prove that
$$frac{1}{(2a+b+c)^2} + frac{1}{(2a+b+c)^2} + frac{1}{(2a+b+c)^2} leqslant frac{3}{16} $$




In the solution given by the official short list solution book (Pg 16), it states that




Without loss of generality, we choose $$a +b+c = 1$$.
Thus, the problem becomes
$$ frac{1}{(1 + a)^2}
+ frac{1}{(1 + b)^2}
+frac{1}{(1 + b)^2}
leqslant frac{3}{16}(frac{1}{a} + frac{1}{b} + frac{1}{c})$$

Applying Jensen’s inequality to the function $$f(x) = frac{x}{ (1+ x)^2}$$
,which is concave for $0 ≤ x ≤ 2$
and increasing for $0 ≤ x ≤ 1$, we obtain
$$α
frac{a}{(1 + a)^2} + β
frac{b}{(1 + b)^2} + γ
frac{c}{(1 + c)^2} leqslant
(α + β + γ)
frac{A}{(1 + A)^2}$$

, where $A =frac{αa + βb + γc}{α + β + γ}.$
Choosing $α =
frac{1}{a}
, β =frac{1}{b},$
and $γ = frac{1}{c}$
, we can apply the harmonic-arithmetic-mean inequality
$$A =frac{3}{frac{1}{a} + frac{1}{b} + frac{1}{c}}

frac{a + b + c
}{3}
=
frac{1}{3}
< 1$$

Finally we prove:
$$ frac{1}{(1 + a)^2}
+ frac{1}{(1 + b)^2}
+frac{1}{(1 + b)^2}
leqslant ( frac{1}{a} + frac{1}{b} + frac{1}{c}) frac{A}{(1 + A)^2} leqslant ( frac{1}{a} + frac{1}{b} + frac{1}{c})frac{A}{(1 + frac{1}{3})^2} = frac{3}{16}(frac{1}{a} + frac{1}{b} + frac{1}{c}) $$




However, Evan Chen's solution (Pg 4) differs by having $a +b+c=3$ and setting $f(x) = frac{1}{16x} - frac{1}{(x+3)^2}$ and allowing Jensen to resolve the rest.



The questions are as follows;




  1. How do you choose before hand which $a + b+ c =$ to choose?


  2. Why particularly does the assumption differ between the 2 solutions?


  3. How does one know before hand the choosing of $α =
    frac{1}{a}
    , β =frac{1}{b},$
    and $γ = frac{1}{c}$?


  4. Where and how did $A =frac{αa + βb + γc}{α + β + γ}$ come from?



Any help would be much appreciated.










share|cite|improve this question
























  • I have found another solution. If you want I am ready to post it.
    – Michael Rozenberg
    Nov 15 at 20:03










  • Here is a problem for me, since $$(a+b+c)({1over a}+{1over b}+{1over c})geq 9$$ we have $a+b+cgeq 3$, so I don't understand either solution.
    – greedoid
    Nov 15 at 20:19










  • @MichaelRozenberg Can you explain please this phenomena?
    – greedoid
    Nov 15 at 20:23










  • @greedoid After homogenization we need to prove that $sumlimits_{cyc}frac{1}{(2a+b+c)^2}leqfrac{3}{16(a+b+c)}sumlimits_{cyc}frac{1}{a}$ and we can assume that even $a+b+c=17$, but we saw that the assuming was $a+b+c=1.$ In your example just $sumlimits_{cyc}frac{1}{a}geq9.$
    – Michael Rozenberg
    Nov 15 at 20:35












  • @greedoid See please the first inequality in my previous post. This inequality is equivalent to the given inequality, but it's homogeneous already.
    – Michael Rozenberg
    Nov 15 at 20:40















up vote
1
down vote

favorite
1












Refer to the problem below (IMO 2009 Shortlist)




Let $a, b, c$ be positive real numbers such that $$frac{1}{a} + frac{1}{b} + frac{1}{c} = a + b+ c$$
Prove that
$$frac{1}{(2a+b+c)^2} + frac{1}{(2a+b+c)^2} + frac{1}{(2a+b+c)^2} leqslant frac{3}{16} $$




In the solution given by the official short list solution book (Pg 16), it states that




Without loss of generality, we choose $$a +b+c = 1$$.
Thus, the problem becomes
$$ frac{1}{(1 + a)^2}
+ frac{1}{(1 + b)^2}
+frac{1}{(1 + b)^2}
leqslant frac{3}{16}(frac{1}{a} + frac{1}{b} + frac{1}{c})$$

Applying Jensen’s inequality to the function $$f(x) = frac{x}{ (1+ x)^2}$$
,which is concave for $0 ≤ x ≤ 2$
and increasing for $0 ≤ x ≤ 1$, we obtain
$$α
frac{a}{(1 + a)^2} + β
frac{b}{(1 + b)^2} + γ
frac{c}{(1 + c)^2} leqslant
(α + β + γ)
frac{A}{(1 + A)^2}$$

, where $A =frac{αa + βb + γc}{α + β + γ}.$
Choosing $α =
frac{1}{a}
, β =frac{1}{b},$
and $γ = frac{1}{c}$
, we can apply the harmonic-arithmetic-mean inequality
$$A =frac{3}{frac{1}{a} + frac{1}{b} + frac{1}{c}}

frac{a + b + c
}{3}
=
frac{1}{3}
< 1$$

Finally we prove:
$$ frac{1}{(1 + a)^2}
+ frac{1}{(1 + b)^2}
+frac{1}{(1 + b)^2}
leqslant ( frac{1}{a} + frac{1}{b} + frac{1}{c}) frac{A}{(1 + A)^2} leqslant ( frac{1}{a} + frac{1}{b} + frac{1}{c})frac{A}{(1 + frac{1}{3})^2} = frac{3}{16}(frac{1}{a} + frac{1}{b} + frac{1}{c}) $$




However, Evan Chen's solution (Pg 4) differs by having $a +b+c=3$ and setting $f(x) = frac{1}{16x} - frac{1}{(x+3)^2}$ and allowing Jensen to resolve the rest.



The questions are as follows;




  1. How do you choose before hand which $a + b+ c =$ to choose?


  2. Why particularly does the assumption differ between the 2 solutions?


  3. How does one know before hand the choosing of $α =
    frac{1}{a}
    , β =frac{1}{b},$
    and $γ = frac{1}{c}$?


  4. Where and how did $A =frac{αa + βb + γc}{α + β + γ}$ come from?



Any help would be much appreciated.










share|cite|improve this question
























  • I have found another solution. If you want I am ready to post it.
    – Michael Rozenberg
    Nov 15 at 20:03










  • Here is a problem for me, since $$(a+b+c)({1over a}+{1over b}+{1over c})geq 9$$ we have $a+b+cgeq 3$, so I don't understand either solution.
    – greedoid
    Nov 15 at 20:19










  • @MichaelRozenberg Can you explain please this phenomena?
    – greedoid
    Nov 15 at 20:23










  • @greedoid After homogenization we need to prove that $sumlimits_{cyc}frac{1}{(2a+b+c)^2}leqfrac{3}{16(a+b+c)}sumlimits_{cyc}frac{1}{a}$ and we can assume that even $a+b+c=17$, but we saw that the assuming was $a+b+c=1.$ In your example just $sumlimits_{cyc}frac{1}{a}geq9.$
    – Michael Rozenberg
    Nov 15 at 20:35












  • @greedoid See please the first inequality in my previous post. This inequality is equivalent to the given inequality, but it's homogeneous already.
    – Michael Rozenberg
    Nov 15 at 20:40













up vote
1
down vote

favorite
1









up vote
1
down vote

favorite
1






1





Refer to the problem below (IMO 2009 Shortlist)




Let $a, b, c$ be positive real numbers such that $$frac{1}{a} + frac{1}{b} + frac{1}{c} = a + b+ c$$
Prove that
$$frac{1}{(2a+b+c)^2} + frac{1}{(2a+b+c)^2} + frac{1}{(2a+b+c)^2} leqslant frac{3}{16} $$




In the solution given by the official short list solution book (Pg 16), it states that




Without loss of generality, we choose $$a +b+c = 1$$.
Thus, the problem becomes
$$ frac{1}{(1 + a)^2}
+ frac{1}{(1 + b)^2}
+frac{1}{(1 + b)^2}
leqslant frac{3}{16}(frac{1}{a} + frac{1}{b} + frac{1}{c})$$

Applying Jensen’s inequality to the function $$f(x) = frac{x}{ (1+ x)^2}$$
,which is concave for $0 ≤ x ≤ 2$
and increasing for $0 ≤ x ≤ 1$, we obtain
$$α
frac{a}{(1 + a)^2} + β
frac{b}{(1 + b)^2} + γ
frac{c}{(1 + c)^2} leqslant
(α + β + γ)
frac{A}{(1 + A)^2}$$

, where $A =frac{αa + βb + γc}{α + β + γ}.$
Choosing $α =
frac{1}{a}
, β =frac{1}{b},$
and $γ = frac{1}{c}$
, we can apply the harmonic-arithmetic-mean inequality
$$A =frac{3}{frac{1}{a} + frac{1}{b} + frac{1}{c}}

frac{a + b + c
}{3}
=
frac{1}{3}
< 1$$

Finally we prove:
$$ frac{1}{(1 + a)^2}
+ frac{1}{(1 + b)^2}
+frac{1}{(1 + b)^2}
leqslant ( frac{1}{a} + frac{1}{b} + frac{1}{c}) frac{A}{(1 + A)^2} leqslant ( frac{1}{a} + frac{1}{b} + frac{1}{c})frac{A}{(1 + frac{1}{3})^2} = frac{3}{16}(frac{1}{a} + frac{1}{b} + frac{1}{c}) $$




However, Evan Chen's solution (Pg 4) differs by having $a +b+c=3$ and setting $f(x) = frac{1}{16x} - frac{1}{(x+3)^2}$ and allowing Jensen to resolve the rest.



The questions are as follows;




  1. How do you choose before hand which $a + b+ c =$ to choose?


  2. Why particularly does the assumption differ between the 2 solutions?


  3. How does one know before hand the choosing of $α =
    frac{1}{a}
    , β =frac{1}{b},$
    and $γ = frac{1}{c}$?


  4. Where and how did $A =frac{αa + βb + γc}{α + β + γ}$ come from?



Any help would be much appreciated.










share|cite|improve this question















Refer to the problem below (IMO 2009 Shortlist)




Let $a, b, c$ be positive real numbers such that $$frac{1}{a} + frac{1}{b} + frac{1}{c} = a + b+ c$$
Prove that
$$frac{1}{(2a+b+c)^2} + frac{1}{(2a+b+c)^2} + frac{1}{(2a+b+c)^2} leqslant frac{3}{16} $$




In the solution given by the official short list solution book (Pg 16), it states that




Without loss of generality, we choose $$a +b+c = 1$$.
Thus, the problem becomes
$$ frac{1}{(1 + a)^2}
+ frac{1}{(1 + b)^2}
+frac{1}{(1 + b)^2}
leqslant frac{3}{16}(frac{1}{a} + frac{1}{b} + frac{1}{c})$$

Applying Jensen’s inequality to the function $$f(x) = frac{x}{ (1+ x)^2}$$
,which is concave for $0 ≤ x ≤ 2$
and increasing for $0 ≤ x ≤ 1$, we obtain
$$α
frac{a}{(1 + a)^2} + β
frac{b}{(1 + b)^2} + γ
frac{c}{(1 + c)^2} leqslant
(α + β + γ)
frac{A}{(1 + A)^2}$$

, where $A =frac{αa + βb + γc}{α + β + γ}.$
Choosing $α =
frac{1}{a}
, β =frac{1}{b},$
and $γ = frac{1}{c}$
, we can apply the harmonic-arithmetic-mean inequality
$$A =frac{3}{frac{1}{a} + frac{1}{b} + frac{1}{c}}

frac{a + b + c
}{3}
=
frac{1}{3}
< 1$$

Finally we prove:
$$ frac{1}{(1 + a)^2}
+ frac{1}{(1 + b)^2}
+frac{1}{(1 + b)^2}
leqslant ( frac{1}{a} + frac{1}{b} + frac{1}{c}) frac{A}{(1 + A)^2} leqslant ( frac{1}{a} + frac{1}{b} + frac{1}{c})frac{A}{(1 + frac{1}{3})^2} = frac{3}{16}(frac{1}{a} + frac{1}{b} + frac{1}{c}) $$




However, Evan Chen's solution (Pg 4) differs by having $a +b+c=3$ and setting $f(x) = frac{1}{16x} - frac{1}{(x+3)^2}$ and allowing Jensen to resolve the rest.



The questions are as follows;




  1. How do you choose before hand which $a + b+ c =$ to choose?


  2. Why particularly does the assumption differ between the 2 solutions?


  3. How does one know before hand the choosing of $α =
    frac{1}{a}
    , β =frac{1}{b},$
    and $γ = frac{1}{c}$?


  4. Where and how did $A =frac{αa + βb + γc}{α + β + γ}$ come from?



Any help would be much appreciated.







inequality contest-math






share|cite|improve this question















share|cite|improve this question













share|cite|improve this question




share|cite|improve this question








edited Nov 16 at 1:48

























asked Nov 15 at 19:45









299792458

236




236












  • I have found another solution. If you want I am ready to post it.
    – Michael Rozenberg
    Nov 15 at 20:03










  • Here is a problem for me, since $$(a+b+c)({1over a}+{1over b}+{1over c})geq 9$$ we have $a+b+cgeq 3$, so I don't understand either solution.
    – greedoid
    Nov 15 at 20:19










  • @MichaelRozenberg Can you explain please this phenomena?
    – greedoid
    Nov 15 at 20:23










  • @greedoid After homogenization we need to prove that $sumlimits_{cyc}frac{1}{(2a+b+c)^2}leqfrac{3}{16(a+b+c)}sumlimits_{cyc}frac{1}{a}$ and we can assume that even $a+b+c=17$, but we saw that the assuming was $a+b+c=1.$ In your example just $sumlimits_{cyc}frac{1}{a}geq9.$
    – Michael Rozenberg
    Nov 15 at 20:35












  • @greedoid See please the first inequality in my previous post. This inequality is equivalent to the given inequality, but it's homogeneous already.
    – Michael Rozenberg
    Nov 15 at 20:40


















  • I have found another solution. If you want I am ready to post it.
    – Michael Rozenberg
    Nov 15 at 20:03










  • Here is a problem for me, since $$(a+b+c)({1over a}+{1over b}+{1over c})geq 9$$ we have $a+b+cgeq 3$, so I don't understand either solution.
    – greedoid
    Nov 15 at 20:19










  • @MichaelRozenberg Can you explain please this phenomena?
    – greedoid
    Nov 15 at 20:23










  • @greedoid After homogenization we need to prove that $sumlimits_{cyc}frac{1}{(2a+b+c)^2}leqfrac{3}{16(a+b+c)}sumlimits_{cyc}frac{1}{a}$ and we can assume that even $a+b+c=17$, but we saw that the assuming was $a+b+c=1.$ In your example just $sumlimits_{cyc}frac{1}{a}geq9.$
    – Michael Rozenberg
    Nov 15 at 20:35












  • @greedoid See please the first inequality in my previous post. This inequality is equivalent to the given inequality, but it's homogeneous already.
    – Michael Rozenberg
    Nov 15 at 20:40
















I have found another solution. If you want I am ready to post it.
– Michael Rozenberg
Nov 15 at 20:03




I have found another solution. If you want I am ready to post it.
– Michael Rozenberg
Nov 15 at 20:03












Here is a problem for me, since $$(a+b+c)({1over a}+{1over b}+{1over c})geq 9$$ we have $a+b+cgeq 3$, so I don't understand either solution.
– greedoid
Nov 15 at 20:19




Here is a problem for me, since $$(a+b+c)({1over a}+{1over b}+{1over c})geq 9$$ we have $a+b+cgeq 3$, so I don't understand either solution.
– greedoid
Nov 15 at 20:19












@MichaelRozenberg Can you explain please this phenomena?
– greedoid
Nov 15 at 20:23




@MichaelRozenberg Can you explain please this phenomena?
– greedoid
Nov 15 at 20:23












@greedoid After homogenization we need to prove that $sumlimits_{cyc}frac{1}{(2a+b+c)^2}leqfrac{3}{16(a+b+c)}sumlimits_{cyc}frac{1}{a}$ and we can assume that even $a+b+c=17$, but we saw that the assuming was $a+b+c=1.$ In your example just $sumlimits_{cyc}frac{1}{a}geq9.$
– Michael Rozenberg
Nov 15 at 20:35






@greedoid After homogenization we need to prove that $sumlimits_{cyc}frac{1}{(2a+b+c)^2}leqfrac{3}{16(a+b+c)}sumlimits_{cyc}frac{1}{a}$ and we can assume that even $a+b+c=17$, but we saw that the assuming was $a+b+c=1.$ In your example just $sumlimits_{cyc}frac{1}{a}geq9.$
– Michael Rozenberg
Nov 15 at 20:35














@greedoid See please the first inequality in my previous post. This inequality is equivalent to the given inequality, but it's homogeneous already.
– Michael Rozenberg
Nov 15 at 20:40




@greedoid See please the first inequality in my previous post. This inequality is equivalent to the given inequality, but it's homogeneous already.
– Michael Rozenberg
Nov 15 at 20:40










2 Answers
2






active

oldest

votes

















up vote
2
down vote














  1. In a homogeneous inequality, this doesn't matter; and $ldots$


  2. $ldots$ there is little difference between the two solutions in this respect, because if you take $a, b, c$ from Problem Shortlist with Solutions [it's on p.14 of the PDF you linked to, by the way, rather than p.16] and write $a' = 3a$, $b' = 3b$, $c' = 3c$, then $a', b', c'$ are the $a, b, c$ of Chen's solution, which homogenises the inequality in exactly the same way.


  3. Given the idea of applying Jensen's inequality to $x/(1 + x)^2$, the choice of weights $alpha = 1/a$, $beta = 1/b$, $gamma = 1/c$ then gives you the LHS of the desired inequality (in its transformed homogeneous form).


  4. The expression $frac{αa + βb + γc}{α + β + γ}$ will naturally appear in any application of Jensen's inequality to a function evaluated at $a, b, c$ with weights $alpha, beta, gamma$.







share|cite|improve this answer





















  • 3. But how do you know before hand (as you were solving the problem for the first time) that the specific choice of weights were as stated? Did it had to do with the initial conditions of $frac{1}{a} + frac{1}{b} +frac{1}{c}$? And how does one derive it from here?
    – 299792458
    Nov 16 at 2:12












  • 4. The same goes for $A$. How do you derive it?
    – 299792458
    Nov 16 at 2:14










  • Unaccountably, I seem to have failed to recognise the important qualification beforehand in both parts 1 and 3 of the question - thus really missing the point of your question altogether! I'm sorry. Also, I don't have any light to shed on how one knows when to apply Jensen's inequality. I don't think I'll go so far as to delete my answer, but I think it is probably best ignored!
    – Calum Gilhooley
    Nov 16 at 2:25


















up vote
0
down vote













Another solution.



We need to show that:
$$frac{3}{16}-sum_{cyc}frac{1}{(2a+b+c)^2}geq0$$ or
$$sum_{cyc}left(frac{3}{16a(a+b+c)}-frac{1}{(2a+b+c)^2}right)geq0$$ or
$$sum_{cyc}frac{(3(2a+b+c)^2-16a(a+b+c))bc}{(2a+b+c)^2}geq0$$ or
$$sum_{cyc}frac{(3b^2+3c^2-6a^2-4ab-4ac+6bc)bc}{(2a+b+c)^2}geq0$$ or



$$sum_{cyc}frac{(2a+3b+3c)((c-a)-(a-b))bc}{(2a+b+c)^2}geq0$$ or
$$sum_{cyc}(a-b)left(frac{(2b+3c+3a)ca}{(2b+c+a)^2}-frac{(2a+3b+3c)bc}{(2a+b+c)^2}right)geq0,$$ which is obvious because
$$(a-b)left((2b+3c+3a)a-(2a+3b+3c)bright)geq0$$ and
$$(a-b)((2a+b+c)^2-(2b+a+c)^2)geq0.$$






share|cite|improve this answer





















    Your Answer





    StackExchange.ifUsing("editor", function () {
    return StackExchange.using("mathjaxEditing", function () {
    StackExchange.MarkdownEditor.creationCallbacks.add(function (editor, postfix) {
    StackExchange.mathjaxEditing.prepareWmdForMathJax(editor, postfix, [["$", "$"], ["\\(","\\)"]]);
    });
    });
    }, "mathjax-editing");

    StackExchange.ready(function() {
    var channelOptions = {
    tags: "".split(" "),
    id: "69"
    };
    initTagRenderer("".split(" "), "".split(" "), channelOptions);

    StackExchange.using("externalEditor", function() {
    // Have to fire editor after snippets, if snippets enabled
    if (StackExchange.settings.snippets.snippetsEnabled) {
    StackExchange.using("snippets", function() {
    createEditor();
    });
    }
    else {
    createEditor();
    }
    });

    function createEditor() {
    StackExchange.prepareEditor({
    heartbeatType: 'answer',
    convertImagesToLinks: true,
    noModals: true,
    showLowRepImageUploadWarning: true,
    reputationToPostImages: 10,
    bindNavPrevention: true,
    postfix: "",
    imageUploader: {
    brandingHtml: "Powered by u003ca class="icon-imgur-white" href="https://imgur.com/"u003eu003c/au003e",
    contentPolicyHtml: "User contributions licensed under u003ca href="https://creativecommons.org/licenses/by-sa/3.0/"u003ecc by-sa 3.0 with attribution requiredu003c/au003e u003ca href="https://stackoverflow.com/legal/content-policy"u003e(content policy)u003c/au003e",
    allowUrls: true
    },
    noCode: true, onDemand: true,
    discardSelector: ".discard-answer"
    ,immediatelyShowMarkdownHelp:true
    });


    }
    });














     

    draft saved


    draft discarded


















    StackExchange.ready(
    function () {
    StackExchange.openid.initPostLogin('.new-post-login', 'https%3a%2f%2fmath.stackexchange.com%2fquestions%2f3000206%2fsignificance-of-assumption-in-competition-inequality-questions%23new-answer', 'question_page');
    }
    );

    Post as a guest















    Required, but never shown

























    2 Answers
    2






    active

    oldest

    votes








    2 Answers
    2






    active

    oldest

    votes









    active

    oldest

    votes






    active

    oldest

    votes








    up vote
    2
    down vote














    1. In a homogeneous inequality, this doesn't matter; and $ldots$


    2. $ldots$ there is little difference between the two solutions in this respect, because if you take $a, b, c$ from Problem Shortlist with Solutions [it's on p.14 of the PDF you linked to, by the way, rather than p.16] and write $a' = 3a$, $b' = 3b$, $c' = 3c$, then $a', b', c'$ are the $a, b, c$ of Chen's solution, which homogenises the inequality in exactly the same way.


    3. Given the idea of applying Jensen's inequality to $x/(1 + x)^2$, the choice of weights $alpha = 1/a$, $beta = 1/b$, $gamma = 1/c$ then gives you the LHS of the desired inequality (in its transformed homogeneous form).


    4. The expression $frac{αa + βb + γc}{α + β + γ}$ will naturally appear in any application of Jensen's inequality to a function evaluated at $a, b, c$ with weights $alpha, beta, gamma$.







    share|cite|improve this answer





















    • 3. But how do you know before hand (as you were solving the problem for the first time) that the specific choice of weights were as stated? Did it had to do with the initial conditions of $frac{1}{a} + frac{1}{b} +frac{1}{c}$? And how does one derive it from here?
      – 299792458
      Nov 16 at 2:12












    • 4. The same goes for $A$. How do you derive it?
      – 299792458
      Nov 16 at 2:14










    • Unaccountably, I seem to have failed to recognise the important qualification beforehand in both parts 1 and 3 of the question - thus really missing the point of your question altogether! I'm sorry. Also, I don't have any light to shed on how one knows when to apply Jensen's inequality. I don't think I'll go so far as to delete my answer, but I think it is probably best ignored!
      – Calum Gilhooley
      Nov 16 at 2:25















    up vote
    2
    down vote














    1. In a homogeneous inequality, this doesn't matter; and $ldots$


    2. $ldots$ there is little difference between the two solutions in this respect, because if you take $a, b, c$ from Problem Shortlist with Solutions [it's on p.14 of the PDF you linked to, by the way, rather than p.16] and write $a' = 3a$, $b' = 3b$, $c' = 3c$, then $a', b', c'$ are the $a, b, c$ of Chen's solution, which homogenises the inequality in exactly the same way.


    3. Given the idea of applying Jensen's inequality to $x/(1 + x)^2$, the choice of weights $alpha = 1/a$, $beta = 1/b$, $gamma = 1/c$ then gives you the LHS of the desired inequality (in its transformed homogeneous form).


    4. The expression $frac{αa + βb + γc}{α + β + γ}$ will naturally appear in any application of Jensen's inequality to a function evaluated at $a, b, c$ with weights $alpha, beta, gamma$.







    share|cite|improve this answer





















    • 3. But how do you know before hand (as you were solving the problem for the first time) that the specific choice of weights were as stated? Did it had to do with the initial conditions of $frac{1}{a} + frac{1}{b} +frac{1}{c}$? And how does one derive it from here?
      – 299792458
      Nov 16 at 2:12












    • 4. The same goes for $A$. How do you derive it?
      – 299792458
      Nov 16 at 2:14










    • Unaccountably, I seem to have failed to recognise the important qualification beforehand in both parts 1 and 3 of the question - thus really missing the point of your question altogether! I'm sorry. Also, I don't have any light to shed on how one knows when to apply Jensen's inequality. I don't think I'll go so far as to delete my answer, but I think it is probably best ignored!
      – Calum Gilhooley
      Nov 16 at 2:25













    up vote
    2
    down vote










    up vote
    2
    down vote










    1. In a homogeneous inequality, this doesn't matter; and $ldots$


    2. $ldots$ there is little difference between the two solutions in this respect, because if you take $a, b, c$ from Problem Shortlist with Solutions [it's on p.14 of the PDF you linked to, by the way, rather than p.16] and write $a' = 3a$, $b' = 3b$, $c' = 3c$, then $a', b', c'$ are the $a, b, c$ of Chen's solution, which homogenises the inequality in exactly the same way.


    3. Given the idea of applying Jensen's inequality to $x/(1 + x)^2$, the choice of weights $alpha = 1/a$, $beta = 1/b$, $gamma = 1/c$ then gives you the LHS of the desired inequality (in its transformed homogeneous form).


    4. The expression $frac{αa + βb + γc}{α + β + γ}$ will naturally appear in any application of Jensen's inequality to a function evaluated at $a, b, c$ with weights $alpha, beta, gamma$.







    share|cite|improve this answer













    1. In a homogeneous inequality, this doesn't matter; and $ldots$


    2. $ldots$ there is little difference between the two solutions in this respect, because if you take $a, b, c$ from Problem Shortlist with Solutions [it's on p.14 of the PDF you linked to, by the way, rather than p.16] and write $a' = 3a$, $b' = 3b$, $c' = 3c$, then $a', b', c'$ are the $a, b, c$ of Chen's solution, which homogenises the inequality in exactly the same way.


    3. Given the idea of applying Jensen's inequality to $x/(1 + x)^2$, the choice of weights $alpha = 1/a$, $beta = 1/b$, $gamma = 1/c$ then gives you the LHS of the desired inequality (in its transformed homogeneous form).


    4. The expression $frac{αa + βb + γc}{α + β + γ}$ will naturally appear in any application of Jensen's inequality to a function evaluated at $a, b, c$ with weights $alpha, beta, gamma$.








    share|cite|improve this answer












    share|cite|improve this answer



    share|cite|improve this answer










    answered Nov 15 at 23:50









    Calum Gilhooley

    4,002529




    4,002529












    • 3. But how do you know before hand (as you were solving the problem for the first time) that the specific choice of weights were as stated? Did it had to do with the initial conditions of $frac{1}{a} + frac{1}{b} +frac{1}{c}$? And how does one derive it from here?
      – 299792458
      Nov 16 at 2:12












    • 4. The same goes for $A$. How do you derive it?
      – 299792458
      Nov 16 at 2:14










    • Unaccountably, I seem to have failed to recognise the important qualification beforehand in both parts 1 and 3 of the question - thus really missing the point of your question altogether! I'm sorry. Also, I don't have any light to shed on how one knows when to apply Jensen's inequality. I don't think I'll go so far as to delete my answer, but I think it is probably best ignored!
      – Calum Gilhooley
      Nov 16 at 2:25


















    • 3. But how do you know before hand (as you were solving the problem for the first time) that the specific choice of weights were as stated? Did it had to do with the initial conditions of $frac{1}{a} + frac{1}{b} +frac{1}{c}$? And how does one derive it from here?
      – 299792458
      Nov 16 at 2:12












    • 4. The same goes for $A$. How do you derive it?
      – 299792458
      Nov 16 at 2:14










    • Unaccountably, I seem to have failed to recognise the important qualification beforehand in both parts 1 and 3 of the question - thus really missing the point of your question altogether! I'm sorry. Also, I don't have any light to shed on how one knows when to apply Jensen's inequality. I don't think I'll go so far as to delete my answer, but I think it is probably best ignored!
      – Calum Gilhooley
      Nov 16 at 2:25
















    3. But how do you know before hand (as you were solving the problem for the first time) that the specific choice of weights were as stated? Did it had to do with the initial conditions of $frac{1}{a} + frac{1}{b} +frac{1}{c}$? And how does one derive it from here?
    – 299792458
    Nov 16 at 2:12






    3. But how do you know before hand (as you were solving the problem for the first time) that the specific choice of weights were as stated? Did it had to do with the initial conditions of $frac{1}{a} + frac{1}{b} +frac{1}{c}$? And how does one derive it from here?
    – 299792458
    Nov 16 at 2:12














    4. The same goes for $A$. How do you derive it?
    – 299792458
    Nov 16 at 2:14




    4. The same goes for $A$. How do you derive it?
    – 299792458
    Nov 16 at 2:14












    Unaccountably, I seem to have failed to recognise the important qualification beforehand in both parts 1 and 3 of the question - thus really missing the point of your question altogether! I'm sorry. Also, I don't have any light to shed on how one knows when to apply Jensen's inequality. I don't think I'll go so far as to delete my answer, but I think it is probably best ignored!
    – Calum Gilhooley
    Nov 16 at 2:25




    Unaccountably, I seem to have failed to recognise the important qualification beforehand in both parts 1 and 3 of the question - thus really missing the point of your question altogether! I'm sorry. Also, I don't have any light to shed on how one knows when to apply Jensen's inequality. I don't think I'll go so far as to delete my answer, but I think it is probably best ignored!
    – Calum Gilhooley
    Nov 16 at 2:25










    up vote
    0
    down vote













    Another solution.



    We need to show that:
    $$frac{3}{16}-sum_{cyc}frac{1}{(2a+b+c)^2}geq0$$ or
    $$sum_{cyc}left(frac{3}{16a(a+b+c)}-frac{1}{(2a+b+c)^2}right)geq0$$ or
    $$sum_{cyc}frac{(3(2a+b+c)^2-16a(a+b+c))bc}{(2a+b+c)^2}geq0$$ or
    $$sum_{cyc}frac{(3b^2+3c^2-6a^2-4ab-4ac+6bc)bc}{(2a+b+c)^2}geq0$$ or



    $$sum_{cyc}frac{(2a+3b+3c)((c-a)-(a-b))bc}{(2a+b+c)^2}geq0$$ or
    $$sum_{cyc}(a-b)left(frac{(2b+3c+3a)ca}{(2b+c+a)^2}-frac{(2a+3b+3c)bc}{(2a+b+c)^2}right)geq0,$$ which is obvious because
    $$(a-b)left((2b+3c+3a)a-(2a+3b+3c)bright)geq0$$ and
    $$(a-b)((2a+b+c)^2-(2b+a+c)^2)geq0.$$






    share|cite|improve this answer

























      up vote
      0
      down vote













      Another solution.



      We need to show that:
      $$frac{3}{16}-sum_{cyc}frac{1}{(2a+b+c)^2}geq0$$ or
      $$sum_{cyc}left(frac{3}{16a(a+b+c)}-frac{1}{(2a+b+c)^2}right)geq0$$ or
      $$sum_{cyc}frac{(3(2a+b+c)^2-16a(a+b+c))bc}{(2a+b+c)^2}geq0$$ or
      $$sum_{cyc}frac{(3b^2+3c^2-6a^2-4ab-4ac+6bc)bc}{(2a+b+c)^2}geq0$$ or



      $$sum_{cyc}frac{(2a+3b+3c)((c-a)-(a-b))bc}{(2a+b+c)^2}geq0$$ or
      $$sum_{cyc}(a-b)left(frac{(2b+3c+3a)ca}{(2b+c+a)^2}-frac{(2a+3b+3c)bc}{(2a+b+c)^2}right)geq0,$$ which is obvious because
      $$(a-b)left((2b+3c+3a)a-(2a+3b+3c)bright)geq0$$ and
      $$(a-b)((2a+b+c)^2-(2b+a+c)^2)geq0.$$






      share|cite|improve this answer























        up vote
        0
        down vote










        up vote
        0
        down vote









        Another solution.



        We need to show that:
        $$frac{3}{16}-sum_{cyc}frac{1}{(2a+b+c)^2}geq0$$ or
        $$sum_{cyc}left(frac{3}{16a(a+b+c)}-frac{1}{(2a+b+c)^2}right)geq0$$ or
        $$sum_{cyc}frac{(3(2a+b+c)^2-16a(a+b+c))bc}{(2a+b+c)^2}geq0$$ or
        $$sum_{cyc}frac{(3b^2+3c^2-6a^2-4ab-4ac+6bc)bc}{(2a+b+c)^2}geq0$$ or



        $$sum_{cyc}frac{(2a+3b+3c)((c-a)-(a-b))bc}{(2a+b+c)^2}geq0$$ or
        $$sum_{cyc}(a-b)left(frac{(2b+3c+3a)ca}{(2b+c+a)^2}-frac{(2a+3b+3c)bc}{(2a+b+c)^2}right)geq0,$$ which is obvious because
        $$(a-b)left((2b+3c+3a)a-(2a+3b+3c)bright)geq0$$ and
        $$(a-b)((2a+b+c)^2-(2b+a+c)^2)geq0.$$






        share|cite|improve this answer












        Another solution.



        We need to show that:
        $$frac{3}{16}-sum_{cyc}frac{1}{(2a+b+c)^2}geq0$$ or
        $$sum_{cyc}left(frac{3}{16a(a+b+c)}-frac{1}{(2a+b+c)^2}right)geq0$$ or
        $$sum_{cyc}frac{(3(2a+b+c)^2-16a(a+b+c))bc}{(2a+b+c)^2}geq0$$ or
        $$sum_{cyc}frac{(3b^2+3c^2-6a^2-4ab-4ac+6bc)bc}{(2a+b+c)^2}geq0$$ or



        $$sum_{cyc}frac{(2a+3b+3c)((c-a)-(a-b))bc}{(2a+b+c)^2}geq0$$ or
        $$sum_{cyc}(a-b)left(frac{(2b+3c+3a)ca}{(2b+c+a)^2}-frac{(2a+3b+3c)bc}{(2a+b+c)^2}right)geq0,$$ which is obvious because
        $$(a-b)left((2b+3c+3a)a-(2a+3b+3c)bright)geq0$$ and
        $$(a-b)((2a+b+c)^2-(2b+a+c)^2)geq0.$$







        share|cite|improve this answer












        share|cite|improve this answer



        share|cite|improve this answer










        answered Nov 16 at 5:19









        Michael Rozenberg

        94.2k1588183




        94.2k1588183






























             

            draft saved


            draft discarded



















































             


            draft saved


            draft discarded














            StackExchange.ready(
            function () {
            StackExchange.openid.initPostLogin('.new-post-login', 'https%3a%2f%2fmath.stackexchange.com%2fquestions%2f3000206%2fsignificance-of-assumption-in-competition-inequality-questions%23new-answer', 'question_page');
            }
            );

            Post as a guest















            Required, but never shown





















































            Required, but never shown














            Required, but never shown












            Required, but never shown







            Required, but never shown

































            Required, but never shown














            Required, but never shown












            Required, but never shown







            Required, but never shown







            Popular posts from this blog

            Quarter-circle Tiles

            build a pushdown automaton that recognizes the reverse language of a given pushdown automaton?

            Mont Emei